Search found 17 matches


A is correct because A explain "diminish wealth", causing by environmental policies.
also, C weaken reasoning; B and D,E is true, but isn't mentioned in premises.

by tailoc

Mon Jul 04, 2011 8:53 am
Forum: Critical Reasoning
Topic: LSAT--Columnist
Replies: 46
Views: 23484

Choose E because another cause

by tailoc

Wed Jun 29, 2011 7:07 am
Forum: Critical Reasoning
Topic: undermines the conclusion
Replies: 110
Views: 46118